1/2 divided by 1 3/7

1/2 Divided By 1 3/7

Answers

Answer 1

Answer:

7/20

Step-by-step explanation:

1/2 divided by 1 3/7

Keep Change Flip

1 3/7=10/7

1/2 x 7/10 = 7/20


Related Questions



Sully built a shelf 8 5/8 inches wide for his DVD collection. If each DVD is 3/8 inches
wide, how many can he fit on the new shelf?

Answers

Answer:

23

Step-by-step explanation:

8 5/8 = 69/8

Each DVD is 3/8 inches

Ignore the denominators

We have 69 and 3

69 is 23 times bigger than 3, so 23 DVD's can fit

Ms. Baranek buys trophies for field day each trophie cost $3.95 she pays $138.25 for all the trophies how many trophies does ms baranek buy

Answers

Answer:

34 trophies

Step-by-step explanation:

1348.25/3.95 = 34.24

Answer:

35

Step-by-step explanation:

138.25 / 3.95 will give you 35

how many roots does the following equation have? x5+x4+x3+x2+x=0?

Answers

Step-by-step explanation:

it has only 1 root

Mark me brainliest

.Learning Task 2: Get the quotient of each item up to the nearest ten thousandths place. Then, round the decimals to their nearest thousandths. Number 1 is done for you.

1.45÷8= 5.6250 -5.625------- example

4.93÷9 = _____ - ______

2. 77÷9 = _____ - ______

5.88 ÷7= _____ - ______

3.81÷7= _____ - ______

6.55÷8=_____ - ______​

sorry if the numbers is a bit confusing

Answers

The responses for the quotient given to the nearest thousandths are

presented in the order of the of the questions as follows;

[tex]Question \ 4) \ \displaystyle \underline{\frac{93}{9} = 10.3333 \approx 10.333}}[/tex]

[tex]Question \ 2) \ \displaystyle \underline{\frac{77}{9} = 8.5555 \approx 8.556}[/tex]

[tex]Question \ 5) \ \displaystyle \underline{\frac{88}{7} = 12.4285 \approx 12.429}[/tex]

[tex]Question \ 3) \ \displaystyle \underline{\frac{81}{7} = 11.5714\approx 11.571}[/tex]

[tex]Question \ 6) \ \displaystyle \underline{\frac{55}{8} = 6.8750\approx 6.875}[/tex]

Reasons:

4. The division is presented as follows;

[tex]\displaystyle \mathbf{\frac{93}{9}}= \frac{90 + 3}{9} = 10 + \frac{1}{3} = 10.\overline 3[/tex]

Given that we have;

[tex]\displaystyle \frac{1}{3} = 0.\frac{10}{3} =0.3 \frac{10}{3} = 0.3333[/tex]

Therefore;

[tex]\displaystyle \underline{\frac{93}{9} = 10.3333 \approx 10.333}[/tex]

2. The division is presented as follows;

[tex]\displaystyle \mathbf{\frac{77}{9}} = \frac{72 + 5}{9} = 8 + \frac{5}{9} = 8. \frac{50}{9} = 8.5\frac{50}{9} = 8.55\frac{50}{9} = 8.555\frac{50}{9} = 8.5555[/tex]

Therefore;

[tex]\displaystyle \underline{\frac{77}{9} = 8.5555 \approx 8.556}[/tex]

5. The division is presented as follows;

[tex]\displaystyle \mathbf{\frac{88}{7}} = \frac{85 + 3}{7} = 12 + \frac{3}{7} = 12. \frac{30}{7} = 12.4\frac{20}{7} = 12.42\frac{60}{7} = 12.428\frac{40}{7} = 12.4285\frac{5}{7}[/tex]

[tex]\displaystyle \underline{\frac{88}{7} = 12.4285 \approx 12.429}[/tex]

3. The division is presented as follows;

[tex]\displaystyle \mathbf{\frac{81}{7}} = \frac{77 + 4}{7} = 11 + \frac{4}{7} = 11. \frac{40}{7} = 11.5\frac{5}{7} = 11.57\frac{10}{7} = 11.571\frac{30}{7} = 11.5714\frac{20}{7}[/tex]

Therefore;

[tex]\displaystyle \underline{\frac{81}{7} = 11.5714\approx 11.571}[/tex]

6. The division is presented as follows;

[tex]\displaystyle \mathbf{\frac{55}{8}} = \frac{48 + 7}{8} = 6 + \frac{7}{8} = 6. \frac{70}{8} = 6.8\frac{60}{8} = 6.87\frac{40}{8} = 6.875\frac{0}{8} =6.8750[/tex]

Therefore;

[tex]\displaystyle \underline{\frac{55}{8} = 6.8750\approx 6.875}[/tex]

Learn more here:

https://brainly.com/question/17913153

Lin's bike travels 100​ meters when her wheels rotate 55​ times. What is the circumference of her wheels? Round your answer to the nearest hundredth.

Thank You!!!

Answers

Answer:

Using formula to calculate the circumference of wheel:

 

where

D is the distance bike travel

n is the number of times rotation

C is the circumference of the wheel

As per the statement:

Lin’s bike travels 100 meters when her wheels rotate 55 times.

here, n = 55 rotation and D = 100 meter.

then by definition:

meter

Therefore, the circumference of her wheel is, 1 9/11

Step-by-step explanation:

Please help me solve this problem ASAP!

Answers

Answer:

im pretty sure the answer is 90°

If A(2, 3) and B(-4, -6), find AB.--

Answers

[tex] \huge \bf༆ Answer ༄[/tex]

The distance between points can be solved using Distance formula (similar to Pythagoras theorem)

[tex] \boxed { \sf D = \sqrt{(x2 - x1) {}^{2} + {(y2 - y1) {}^{2} } }}[/tex]

Put the values : -

[tex] \sf D = \sqrt{ (2 - ( - 4)) {}^{2} + (3 - ( - 6)) {}^{2} } [/tex]

[tex] \sf \: D = \sqrt{ {(6)}^{2} + (9) {}^{2} } [/tex]

[tex] \sf D = \sqrt{36 + 81} [/tex]

[tex] \sf D = \sqrt{117} [/tex]

[tex] \sf D = 3 \sqrt{13} [/tex]

Distance between the points = 3√13 units

[tex]꧁ \: \large \frak{Eternal \: Being } \: ꧂[/tex]

Answer:

If you are trying to find the line, simply use a graph.

Step-by-step explanation:

Answer is D)-4 but please explain how to get it

Answers

Answer:

D

Step-by-step explanation:

We could plug x into the equation but to make it easier we simplify it first

[tex]\frac{1}{x} -\frac{1}{x+1}[/tex]

[tex]-\frac{1}{x+1} + \frac{1}{x}[/tex]

[tex]-\frac{1}{2x+1}[/tex]

x = -1/2

-2-2 = -4

What number is furthest from 0 on a number line? 16 or 18?

Answers

18. _______________________________________

Simplify.

x2+2x−3/
x−4/
2x2+5x−3/
x^2−16



x2−x−12/2x−3

x2−5x−4/2x−1

x2+3x−4/2x−1

x2+x−12/2x−3

Answers

Answer:

x^2 + 7x- 3/x - 3/x^2 -16

Step-by-step explanation:

Key:  ( ^ ) means exponent

= x^2 + 2x- 3/x -2x^2 + 5x- 3/x^2 -16

= x^2 - 2x^2 + 2x+5x - 3/x- 3/x^2 -16

= x^2 - 2x^2 = -x^2

=  -x^2 + 2x + 5x- 3/x- 3/x^2-16

As you see you add 2x and the 5x and got 7x

= Answer : x^2 + 7x- 3/x - 3/x^2 -16

Answer:

I think the answer is x^2+3x-4/2x-1

Write the equation of the following parabolae in their canonical form and hence find their vertices, foci and directrix.
1.
[tex] {y}^{2} - 6y - 2x + 19 = 0[/tex]
2.
[tex] {x}^{2} + 4x + 4y + 16 = 0[/tex]

Answers

#1

Given:

y² - 6y - 2x + 19 = 0

This is a horizontal parabola.

Canonical form of horizontal parabola is:

4a(x - h) = (y - k)², where (h, k) is vertex

Focus is:

F(h + a, k)

Directrix is:

x = h - a

Convert the equation:

y² - 6y - 2x + 19 = 0y² - 6y  + 9 - 2x + 10 = 0(y - 3)² = 2x - 10(y - 3)² = 4(1/2)(x - 5)

We got:

h = 5, k = 3, a = 1/2

Focus is:

F(5 + 1/2, 3) = F(5.5, 3)

Directrix is:

x = 5 - 1/2 = 4.5#2

Given:

x² +4x + 4y + 16 = 0

This is a vertical parabola.

Canonical form of vertical parabola is:

4a(y - k) = (x - h)², where (h, k) is vertex

Focus is:

(h, k + a)

Directrix is:

y = k - a

Convert the equation:

x² + 4x + 4y + 16 = 0x² +4x + 4 + 4y + 12 = 0(x + 2)² = -4y - 12(x + 2)² = 4(-1) (y + 3)

We got:

h = - 2, k = -3, a = - 1

Focus is:

F(-2, -3 - 1) = F(-2, - 4)

Directrix is:

y = -3 - (-1) = - 2

Em uma avaliação que vale até 5 pontos, João tirou 4,2 pontos. Quanto João teria tirado caso a prova valesse 10 pontos?

Answers

Answer:

8.4 pontos

Step-by-step explanation:

4.2/5

multiplique o numerador e o denominador por 2

8.4/10

PLS HELP !!! i’ll mark you for 1st right answer

Answers

Answer:

130 grams

Step-by-step explanation:

Add the second amount which equals 150, and then subtract by 20.

ACCURATE ANSWER=BRAINLIST
The first four terms of a sequence are shown.

7, 25, 43, 61, ...
Which of the following expressions can be used to find the nth term of the sequence?

Question 3 options:

7+18n

18+7n

18+7(n−1)

7+18(n−1)

Answers

Answer:

7+18n

might be the answer sorry if I am wrong

It’ be 7 + 18(n-1)

I plugged in the fourth term for n, and only that expression gave the correct answer of 61

please help with statistic 20

Answers

Part A

p = 0.32 = probability of someone voting by internet

n = 13 = sample size

x = 10 = number of people who voted by internet

Apply the binomial probability formula.

[tex]P(x) = (_n C _x)*(p)^{x}*(1-p)^{n-x}\\\\P(10) = (_{13} C _{10})*(0.32)^{10}*(1-0.32)^{13-10}\\\\P(10) = (286)*(0.32)^{10}*(0.68)^{3}\\\\P(10) \approx 0.0010124942242\\\\P(10) \approx \textbf{0.00101}\\\\[/tex]

Note: the [tex]_n C_x[/tex] refers to the combination formula.

Answer:   0.00101

=============================================================

Part B

We could apply the binomial theorem for the following values of x: {10,11,12,13}. Then would add up the results.

However, such a task is fairly tedious and it's more efficient to use computer software (or a graphing calculator). Also, there's the element of limited time that you'll need to consider. I showed the steps for part A above because your teacher may have wanted you to list them. Usually it's better to rely on software for that task as well. At the very least, it's useful to have software to check your answer.

Whichever method you use, you should find the following:

P(10) = 0.00101P(11) = 0.00013P(12) = 0.00001P(13) = 0

The last value isn't exactly 0, but it's so small that it's effectively zero. This is because we only have 5 decimal places to work with.

Add up those four values to get:

[tex]P(x \ge 10) = P(10)+P(11)+P(12)+P(13)\\\\P(x \ge 10) \approx 0.00101+0.00013+0.00001+0\\\\P(x \ge 10) \approx 0.00115\\\\[/tex]

A faster shortcut is to use your calculator's binomCDF function.

This result of 0.00115 is fairly small. The usual distinction we make between whether a probability is small or not is to set up some kind of threshold. This threshold is known as the significance level. By default, it's set to 0.05 unless your teacher specifies otherwise. Another common significance level is 0.01, but it's usually mentioned.

Since 0.00115 is smaller than 0.05, and even 0.01, we consider the probability of getting 10 or more internet voters to be a significant event. It's fairly unusual to get 10 or more people voting by internet in this sample of n = 13 people.

Answer: Choice A) Yes, because the probability of 10 or more is 0.00115, which is low.

=============================================================

Part C

Like with part B, we could find the binomial probability values for x = {1,2,3,...,11,12,13}. We could add up the values like I did, or use the binomCDF function which is faster.

Another approach is to use the complement of the event "at least 1". If x is some number in the set {0,1,2,..,12,13}, then either [tex]x = 0[/tex] or [tex]x \ge 1[/tex]

From this, we can say

[tex]P(x=0) + P(x \ge 1) = 1\\\\P(x \ge 1) = 1-P(x=0)\\\\P(x \ge 1) \approx 1-0.00665\\\\P(x \ge 1) \approx 0.99335\\\\[/tex]

This then rounds to 0.993

Answer:  0.993

Can you help me on this question Plss??

Answers

it’s the second choice

Guided Practice

Type your answer and then click or tap Done.
Use the ^ symbol (Shift+6 on the keyboard) to signify an exponent. For example, for 2 to the third power, enter 2^3.

Identify the quadratic term in the function.

f(x)=2x^2−3x+5

Answers

Answer:

2x^2

Step-by-step explanation:

Q:

identify the quadratic term in the function f(x)=2x^2-3x+5

A:

The quadratic term is 2x^2.

Answer:

2x2-3x-5=0

Step-by-step explanation:

(2x2 -  3x) -  5  = 0

Answer:

The quadratic term in the function is  

Step-by-step explanation:

Given : Function  

To find : Identify the quadratic term in the function ?

Solution :

The quadratic  form of the function is  

Where,  is quadratic term.

On comparing with given function,  

is the quadratic term.

Therefore, the quadratic term in the function is

A store owner has 65.5 pounds of candy. If he splits the candy equally into 5 boxes, how many pounds of candy will each box contain?

Answers

Answer: Each box of candy would contain 13.1 pounds of candy.

Step-by-step explanation:

Divide 65.5 by 5. Hope this helps!

The algebraic form of an with no solution is?

1. a = a
2. x = a
3. a = b


Which one is the answer?

Answers

a = b

In algebra, X normally correlated with a specific variable so if x = anything, (x = 4) that is a real equation. a = b just means a number equal to another number (3 = 4) which is not true.

plz help this is past due

Answers

Answer:

-8

Step-by-step explanation:

if you sub in 8 as the value for y, you get

-4x - 8 = 24

add 8 on both sides to simplify

-4x = 32

divide both sides by -4

x = -8

How many times does 75 fit into 250?

Answers

Answer:

3.333333 times

Step-by-step explanation:

Answer:

just about 3 times

Step-by-step explanation:

250 / 75

Work out Sarah's total pay for the last week

Answers

Answer:

190

Step-by-step explanation:

Attached picture...

A) (2x-10)(3x+1)
B) (3x+2)(2x-5)
C) (2x+5)(3×+1)
D) (2x+1)(3×+5)
E) None of the above. The quadratic f(x) = 6x] + 11X - 10
would have which of the following
factors?

Answers

Answer:

B

Step-by-step explanation:

(3x+2)(2x-5)

Brainliest?

Jessica and her brother collects stamps. When Jessica gives her brother 1% of her stamps, she has 2,475 stamps left. Calculate how many stamps Jessica had originally.

Answers

Answer:

2500

Step-by-step explanation:

2475 stamps is 99% of her original number of stamps.

SO,

[tex]2475 \div 99\% = 2500[/tex]

She originally had 2500 stamps

please help me I don't understand what it is asking

Answers

Answer:

3.5 ft

Step-by-step explanation:

[tex]1 \frac{3}{4} = \frac{7}{4} [/tex]

7/4 inch = ? feet

[tex] \frac{ \frac{1}{2} }{ \frac{7}{4} } = \frac{1}{x} \\ \\ \frac{4}{14} = \frac{1}{x} \\ x = \frac{7}{2} [/tex]

3.5 ft

Answer:

3.5 ft.

Step-by-step explanation:

The question describes a bookshelf with width given in inches and we have to find the actual width in feet.

That makes it

Conversion of lengths from inches to feets:

The bookshelf drawn by Sarita is

[tex]1 \frac{3}{4} [/tex]

inches wide.

Converting a mixed fraction to its simplest form

Any fraction with it's quotient greater than or equal to 1 can be expressed in the form of mixed fraction.

It combines a whole number and a fractional reminder. The whole number represents how many complete wholes you have.

So, to convert a mixed fraction to an improper fraction, following steps will be conducted:

STEP I

(multiply the whole number by the denominator)

In this case,

[tex] 1\frac{3}{4} [/tex]

Denominator = 4whole number = 1

= 4 × 1

= 4

STEP II

(add the product to the numerator)

numerator = 3 product obtained din step 1 = 4

= 3 + 4

= 7

STEP III

(place the numerator obtained in Step 3 over the denominator)

new numerator = 7 denominator = 4

Which gives us the fraction:

[tex] \boxed{ \frac{7}{4} }[/tex]

So, the width of the bookshelf is 7/ 4 in.

Expressing it in feets:

[tex] \mathsf{ \frac{1}{2} \: inches \: represent = 1 \: feet }[/tex]

[tex] \mathsf{ 1 \: inch \:will\: represent = 1 \times \: \frac{2}{1} feet }[/tex]

[tex] \mathsf{ \frac{7}{4} \: inches \:will\: represent = 2 \times \: \frac{7}{4} feet }[/tex]

[tex] \implies \mathsf{ \underline{\frac{7}{2} \: ft. }}[/tex]

[tex] \mathsf{or}[/tex]

[tex] \implies \: \mathsf{ \underline{3.5 \: ft.}}[/tex]

That makes it Option: D) 3.5 ft.

show me how to do the work please

Answers

[tex]\large\boxed{\mathsf{\dfrac{x}{-4}\leq 12}}\\\\\large\text{MULTIPLY -4 to BOTH SIDES}\\\\\large\boxed{\mathsf{-4\times -\dfrac{1}{4}x \leq -4 \times 12}}\\\\\large\text{CANCEL out: }\rm{-4 \times-\dfrac{1}{4}}\large\text{ because it gives you 1}\\\\\large\text{KEEP: }\rm{-4\times 12}\large\text{ because it helps solve for what is being compared to the }\\\large\text{x-value}\\\\\large\text{NEW EQUATION: }\rm{x \geq\ -4 \times 12}\\\\\large\text{SIMPLIFY IT}\uparrow[/tex][tex]\huge\text{Therefore, your answer is: \boxed{\mathsf{x\geq-48}}}\huge\checkmark[/tex]

[tex]\large\boxed{\mathsf{It\ is\ an\ CLOSED\ circle\ shaded\ to\ the\ right}}[/tex]

[tex]\huge\text{Good luck on your assignment \& enjoy your day!}[/tex]

~[tex]\frak{Amphitrite1040:)}[/tex]

5 peppers for $5.50 how much is each

Answers

Answer:

$1.10

Step-by-step explanation:

5.50 divided by 5, take a way the decimal in order to divide. 550/5=110

55 divided by 5 is 11 just add a zero

what is the solution to the system of equations​

Answers

Here the answer hope this help

Finding slope from tables and

Answers

Answer:

m=-5/2 b=2

Step-by-step explanation:

It goes over 2 times ever time it goes down 5 times. and when the y is 0 the x is 2

please help me with this math problem

Answers

Step-by-step explanation:

the group of x friends is SPLITTING the cost of $50. Therefore to find the amount each person pays after the split, you will divide 50 by the amount fo friends (x). Each person is also paying an extra $5 for dessert. Therefore, you will be adding that 5 to everyone's total after splitting.

So the equation you will get is

[tex]a(x) = \frac{50}{x} + 5[/tex]

Answer:

A(x)= (50/x)+5

Step-by-step explanation:

50 bucks divided by the number of people in the group which is X and each must also pay 5 bucks for dessert.

Other Questions
A wind turbine has an efficiency of 0.7. The wind provides 8000 J of kinetic energy to the blades of the wind turbine. How much useful energy will the wind turbine transfer? Use the equation below to help you. You may need to rearrange the equation. Discuss the likely impact on papua new guinea if its copper and gold mines become exhausted? Describe the evidence of a geologic process that you observed. can any equation in point-slope form also be written in slope-intercept form? What percent of all farms in the US are small family farms?A.30%B. 50%C. 70%D. 90% your reaction to the one thing that surprised you the most about food waste? (8 7 3) 7 Match each expression to another expression with the same value Situational depression can occur when bad things happen.True or false are 15/18 and 10:12 equal ratios Alex owns a miniature model of a train. The actual train is 14 feet tall and 70 feet long. If Alex's model is 6 inches tall, what is its length, rounded to the nearest inch?. how do i find the answer In other parts of the country speeding may be a federal offence.What is wrong with this sentence? 1/4 xy if x = -2/3 and y = 3/5 in figure abcd is a parallelogram DE is perpendicular to AB DF perpendicular to BC and angle EDF=60 degree find angles of parallelogram Explain why ferns do not produce flowers? A Ferns produce single reproductive cells called spores. B Ferns can only grow in colder places where flowers cannot survive. C Ferns do no make need fertilized egg cells. D Ferns do not need to attract pollinators. SOMEONE PLEASE HELPILL MARK BRAINLYEST WHAT EVER IT IS CALLED PLEASE 1. Find the value of x in the figure below if GH is parallel to JK.A. 4 unitsB. 9.8 units C. 6.7 units D. 5 units 1/4x^3 + 3w - x; if w = -12 and x = 10 50 Points Please Help Me for Physics Briefly describe V-T graph For every point please if its increasing if its flat or if its positive or negative help fast plz help 25 pts and ill mark u brainliest What is the value of x?